« first day (43 days earlier)      last day (531 days later) » 
00:00 - 16:0016:00 - 00:00

12:00 AM
0
Q: Angles between curves

gopher42Question: Find the angle between the curves y = sin 5x and y = cos 5x. I have used the formula tan θ = absolute((m1-m2)/1+(m1 * m2)) and found the gradient values by deriving each function. I get an angle of 27.124 degrees. But the answer in the book says : 31/6 degrees ?

0
Q: Spectral radius in Banach algebra is commutative

kissanpentuI want to show that for a Banach algebra $A$ and elements $x,y \in A$, we have $$ r_A(xy) = r_A(yx), $$ where $r_A$ is a spectral radius. This is how I am trying to do that: $$ r_a(xy) = \lim_{n \rightarrow \infty} \| (xy)^n \| ^{\frac1n}=\lim_{n\rightarrow \infty} \|x(yx)^{n-1} y \|^{\frac1n}. $...

Welcome to Math.SE, gopher42. This site uses MathJax formatting of formulas. More tips here. (autocomment)Normal Human 21 secs ago
0
Q: Real analysis true/false

Matt G(1) Let $f$ be a continuous function on $[0,1]$. Then for every partition $P$ of $[0,1]$, the lower and upper Riemann sums of $f$ over $P$ satisfy $L(f,P)\neq U(f,P)$. (2) Let $f:[0,\infty)\to\mathbb{R}$ be a continuous function such that $\lim_{x\to\infty}f(x)=0$. Then $f$ has a maximum value o...

0
Q: Is there an obvious reason why the number of binary Lyndon words is equal to the number of irreducible polynomials over GF(2)?

Geoffrey CritzerThe title of Sloane's A001037 is: Number of degree-n irreducible polynomials over GF(2); number of n-bead necklaces with beads of 2 colors when turning over is not allowed and with primitive period n; number of binary Lyndon words of length n. The first few terms of the sequence are (for n = 1,2,...

This site uses MathJax formatting of formulas. More tips here. (from a bot)Normal Human 21 secs ago
0
Q: Logical disjunctions question

JanzenAre the formulas that make up a disjunction called conjuncts? I am new to logic and need to know this for an assignment.

Welcome to Math.SE, Janzen. Words such as question are uninformative in titles. Please edit the title so that it better describes the specifics of your question. Do not hesitate to make it longer or include a formula if needed. More tips here. (autocomment)Normal Human 21 secs ago
0
Q: Discussion: An interesting was to visualize the Mandelbrot Set

Jerry GuernThis is a discussion opener about something neat I found. I put some specific questions in there, but please chime in with any interesting insights. I have noticed some interesting properties of Mandelbot series that lead to a different way to plot the M-Set to elucidates certain details and pr...

0
Q: Typical sequences and entropy

edwineveningfallThe book "Probability, Random Processes, and Statistical Analysis" (written by Hisashi Kobayashi and Brian L. Mark and William Turin), talks about the role of entropy in characterising typical sequences (page 257). It says that in a coin tossing experiment: When we change the experiment of fa...

 
12:22 AM
0
Q: Два принятых ответа

SuvitrufНаткнулся тут на вопрос: Как удалить элемент из выпадающего списка с помощью jQuery? И в нём два принятых ответа. Может кто-то объяснить сей феномен?

-1
Q: Can I get more hats by asking stupid questions on stackoverflow meta?

Xan-Kun Clark-DavisWhat I tried so far: See above. What does not work: Still not enough hats. Expected behaviour: Looking really cool in the best hat ever. Full stack trace: FAILED: OnHeadAdditionalApparel java.lang.OnHeadAdditionalApparelMissingException at com.body.head.scalp.DefaultHatProxyFactory.assertInd...

 
0
Q: Image of unit sphere being hyper ellipse proof (SVD)

Amadeus BachmannWhen I check for the proof of singular value decomposition, they all assume the following is true: The image of the unit sphere under any $m * n$ matrix is a hyper ellipse. However I could not find a decent proof for this, even though I googled for hours. Maybe I am using wrong keywords. Could ...

0
Q: Problem in Combinatorics

AlexaConsider $n \leq 2k$ and $A_1,...,A_m$ is a family of $k$-element subsets of $[n]$ such that $A_i \cup A_j \neq [n] \forall i,j \in [m].$ I want to show that $m$ is bounded above by $(1-\frac{k}{n})\binom{n}{k}$ I understand that the Erd\H{o}s-Ko-Rado theorem says that if $2k \leq n$ then every ...

Short title. Title contains problem. Problem in Combinatorics
0
Q: Prove L=0 when improper integral of f(x) converges and lim f(x) =L as x --> infinty

ddbb1994Here is the question. If $\int ^\infty _1 f(x)dx$ converges and $\lim_{x\to \infty} f(x)=L$, prove that $L=0$. Any help would be appreciated.

 
0
Q: Possible alternatives to repeatedly commenting

404UserNotFound(See also here, but this is on a much shorter timespan.) Today, I just had a couple of experiences with using comments for this question I just asked that makes me wonder if answers could be used to clarify other answers, if it's better to make an edit in the question to respond to that answer, ...

 
0
Q: Setting up a polynomial that has a wiggle root at 0?

ShantyI am trying to set up a polynomial such that it has a wiggle root at 0. I have gotten it such that there are wiggle roots at -1 and 1. My question is: 1. What part of the polynomial determines whether the root is a wiggle root or not? and 2. How do I make that change in my current polynomial. Th...

This site uses MathJax formatting of formulas. More tips here. (from a bot)Normal Human 21 secs ago
0
Q: Verify proof of $ f(x)=e^x $ if $ f(x+y)=f(x)f(y) $ and $ f'(x)$ exists for all $x$

Imre DeákThis is exercise 6.26.8 from Tom Apostol's Calculus I, I'd like to ask someone to verify my proof. I'd be also interested in alternative proofs: If $ f(x+y)=f(x)f(y) $ for all $ x $ and $ y $ and if $ f(x)=1+xg(x) $, where $ g(x) \to 1 $ as $ x \to 0 $, prove that (a) $f'(x)$ exists for every $x...

0
Q: $X:||A||_2<1 \iff matrix \begin{bmatrix} I&X^*\\X&I\\\end{bmatrix} $ is positive

alevFollowing question seems so simple, yet I could not come up with a solution. I started to think that there might be sth wrong with the question. Could you please take a look? For a matrix $X:||X||_2<1 \iff matrix \begin{bmatrix} I&X^*\\X&I\\\end{bmatrix} $ is positive

0
Q: Prove gcd(a,c)=gcd(a,b)=1

JimIf a,b,c are contained in Z, gcd(a,b)=1 and c|(a+b) Then prove gcd(a,c)=gcd(b,c)=1 I think this can be proven with linear combinations but I'm not sure how to go about starting the proof.

Short title. Title ends with a digit. Prove gcd(a,c)=gcd(a,b)=1
0
Q: Invertible elements in Banach algebra.

kissanpentuSuppose $A$ is a Banach algebra. Is it true that a) $x$ and $xy$ are invertible, then so is $yx$; b) $xy$ and $yx $ are invertible, then so are $x$ and $y$?

 
1:02 AM
0
Q: Arc length of the squircle

Ben LongoThe squircle is given by the equation $x^4+y^4=r^4$. Apparently, its circumference or arc length $c$ is given by $$c=-\frac{\sqrt[4]{3} r G_{5,5}^{5,5}\left(1\left| \begin{array}{c} \frac{1}{3},\frac{2}{3},\frac{5}{6},1,\frac{4}{3} \\ \frac{1}{12},\frac{5}{12},\frac{7}{12},\frac{3}{4},\frac{1...

0
Q: Congruence Substitutions

John L.If I'm asked to calculate $319^{566} \mod(23)$, and I know $319 \equiv 20 \mod(23)$, is it mathematically to correct to then calculate $20^{566}$ instead? I feel the answer is yes, but I've an exam tomorrow and would rather a concrete notion rather than an inclination. Thank you guys!

 
1:23 AM
0
Q: Is this geometry question about a pentagon correct?

Puzzled417Problem In pentagon $ABCDE$, $AB=BC=2,CD=\sqrt{2}$,and $EA= \sqrt{3}$. If $\angle{A}=90^{\circ}$, and $\angle{B} = 120^{\circ}$, what is the area of $ABCDE$? I just need some reaffirmation that there is no solution to this problem. There are infinitely many pentagons with the properties in ...

Words such as question are uninformative in titles. Please edit the title so that it better describes the specifics of your question. Do not hesitate to make it longer or include a formula if needed. More tips here. (autocomment)Normal Human 20 secs ago
 
0
Q: Is there a secret Jon Skeet Hat?

Tristan WileySince there's pleanty of secret hats, is there a Jon Skeet hat? Since, Jon Skeet is amazing Jon Skeet coded his last project entirely in Microsoft Paint, just for the challenge. And Jon Skeet is immutable. If something's going to change, it's going to have to be the rest of the universe.

 
0
Q: Graphs and Matrices

GeorgeI'm starting to prove some of the major relations between graph problems and linear algebra. Take $G$ to be an undirected graph, and $A$ as its adjacency matrix. Let $\bar{d}$ be the average degree of the graph and $I$ to be the max degree of $G.$ I want to show that the largest eigenvalue $\ti...

Short title. Graphs and Matrices
 
1:46 AM
0
Q: Show diag($BA$)=$B$ diag($A$)

Melissa HerringtonSuppose $B$ is an $n \times n$ diagonal matrix. How to show that for any $n \times n$ matrix A, diag($BA$)=$B$ diag($A$)

Short title. Short question. Show diag($BA$)=$B$ diag($A$)
0
Q: Let $(M,d)$ be a metric space and $X \subset M$ a discrete subset.

SantosThen, for each $x \in X$, there exists $B_x = B(x;r_x)$ such that $x \neq y \Rightarrow B_x\cap B_y = \emptyset$. My attempt For each $x\in X$, let $r_x = \inf\{ d(x,a): a\in X-\{x\}\}$. If we suppose $r_x = 0$, then $\forall \varepsilon>0$, $\exists$ $ a \in X-\{x\} : d(x,a) < \varepsilon$. ...

0
Q: f : R → R satisfies: (i) f(2) = 5; (ii) f(3) = 8; (iii) f'(x) ≤ 4 $sin^2$ πx for all x ∈ R. How many values of s ≥ 2 satisfy f(s) = $s^2$ ?

CrescentNAlso f is differentiable. I could conclude that x < 3 by the 3rd condition, and tried to use the Mean Value Theorem but found it hard to go on. Can anyone give some hints? Thanks!!

 
2:08 AM
0
Q: Exponent help - adding and subtracting

queens718I know 13^5 + 13^3 = 169 but can someone please explain to me step by step how to do this? How does this happen? 13^3 x 13^2 + 13^3 (13^2 + 1) -> 13^2 = 169

Words such as help are uninformative in titles. Please edit the title so that it better describes the specifics of your question. Do not hesitate to make it longer or include a formula if needed. This site uses MathJax formatting of formulas. Questions tend to get more attention when they have a tag for a broad area of mathematics relevant to the question. Some of these tags might fit. More tips here. (from a bot)Normal Human 21 secs ago
0
Q: standard deviation/mean of test result question

heiopThere is a general achievement test that I have taken and here is a screenshot of how they says it is calculated: http://i.imgur.com/gNUxa62.png 'On a scale of 0 - 50, with a mean of 30 and a standard deviation of 7', what would the standardised score be if for example I got 35/40 for one of the...

Welcome to Math.SE, heiop. Words such as question are uninformative in titles. Please edit the title so that it better describes the specifics of your question. Do not hesitate to make it longer or include a formula if needed. More tips here. (autocomment)Normal Human 21 secs ago
0
Q: Finding the order of an integer n modulo a prime given a congruence equation

user286417Let $m$ be some prime greater than or equal to $5$. There exists an $n$ such that $n^2+n+1 \equiv 0 \mod m$ Find the order of $n$ modulo $m$, and show $m \equiv 1 \mod 3$

 
2:36 AM
0
Q: How to deal with identical questions but only one being closed

WilliamDiamond Moderators and others(my bounty was given back). I believe both this question and this one should be closed not just one. I want a justification and a technique to better help users. I am tired of finding nearly identical questions(which people use as justifications to ask new question...

 
0
Q: Proving that a function is injective

SarahLet X and Y be sets and let f: X-->Y be a function between them. Suppose that U1, U2 are subsets of X. Show that if f is injective then f[(U1 union U2)^c]= f[U1^c] intersect f[U2^c] I'm not exactly sure how to go about this proof. My first thought was that the left hand side is De Morgans Law b...

Tagged proof-explanation. Proving that a function is injective
 
you delete your meta.SE?
 
2:55 AM
1
Q: Why both options were enabled in first post review at today's morning?

manetsusI have seen a peculiar thing while reviewing first post at today's morning. Could you observe something different? Ow, No Action Needed and I'm Done, both the options were enabled simultaneously! Is this as usual? I think not. Because I had never seen such type of scenario before. Could a...

0
Q: Whats the process for get all the secret hats!

ElenasysI have tried all the years to get all the secret hats with no success!, Whats the process! How could i get all the hats!!! thanks in advance

 
0
Q: Prove: if lim x-->a of f(x) = lim x--> a of g(x) = infinity, then lim x-->a of [f(x) = g(x)] = infinity

AnalysisProofif lim x-->a of f(x) = infinity, then we know: for all M>0, there exists D1 such that if |x-a| < D1, then f(x) > M/2 if lim x--> a of g(x) = infinity, then we know: for all M>0, there exists D2 such that if |x-a| < D2, then g(x) > M/2 So for all positive M choose D = max{D1, D2} Then if |x - a| <

This site uses MathJax formatting of formulas. Tag (proof-verification) should not be the only tag a question has. Please add a tag for a subject area to which the question belongs. More tips here. (autocomment)Normal Human 21 secs ago
0
Q: Wondering if something is an algebra. If it is, question about closure under complements.

majmunIf we define $$ $$\mathcal{J} = \{\text{ all intervals contained in [0,1]}\} $$ Then $$ B_0 = \{ \text{ all finite unions of elements of }\mathcal{J}\} $$ Is an algebra (so it is closed under the formation of complements (but it is not a $\sigma$ algebra) . Now consider $$ B_1 = \{\text{ all fin...

 
3:29 AM
0
Q: Are you into spreading Winter Bash happiness?

PetahChristianOne of the unexpected joys of Winter Bash has been contributing to help SO members to earn secret hats. Have you taken the opportunity to give another SO user the gift of a hat?

 
3:40 AM
0
Q: I mustache you a question...

hellyaleDo you all like my mustache? Can someone make a mustache tag for me please? Winter bash just is not complete without one in my opinion. Would make 2015 complete for me.

 
3:58 AM
0
Q: Expanding a function as an eigenfunction

user286417Expand the function $U(x,t) = 1$ as a eigenfunction expansion of $cos(\frac{ 2n+1}{ 2} )πx$ for $n ≥ 0$. I missed my last ODE class and am having trouble with this problem. Can anyone be of help?

Tagged pde, differential-equations. Expanding a function as an eigenfunction
 
4:14 AM
0
Q: Awarded extra 'fanatic' badges

Ben S.I am just curious to know as to why I have been awarded three 'fanatic' badges, all at the same time? I am aware that I have earned one, but I am curious if the other two are the result of some sort of bug or if this is meant to happen. Thank you!

1
Q: Information Request - Number of new accounts that have their first question closed shortly after account creation?

AndyIn the Tavern, the topic of post quality came up recently. One of the ideas thrown out was to delay the ability of newly registered users from posting immediately - perhaps a 30 minute delay to 'encourage' the new user to do just a bit more research before filling out the textboxes. Thus, my requ...

0
Q: Выборы и локализация

QwertiyНа странице выдвижения кандидата поясняющий текст сформулирован странно: Хорошие модераторы часто: ► иметь достаточно высокую репутацию, указывающую на активное участие ► интересоваться развитием сообщества на Мете ► подавать пример, демонстрируя терпение и уважением к другим участникам...

 
0
Q: Calculate $\sum_{n=0}^\infty$ $(n+1)(n+2)(\frac{i}{2})^{n-1}$

JAEMTO$\sum_{n=0}^\infty$ $(n+1)(n+2)(\frac{i}{2})^{n-1}$ I tried to separate real number(n=0,2,4...) and complex number that is not a real number(n=1,3,5,...) But it didn't work. So I did another way; Using Cauchy's integral thm: Let $f(z)=(\frac{z}{2})^{n+2}$ Then $4f''(i)$= $(n+1)(n+2)(\frac...

0
Q: specify function given certain constraints

user159452Let $F:V\rightarrow{}\mathbb{R}^{+}_{0}$ be a function on domain $V=\{(x_{1},x_{2},x_{3},x_{4})|(x_{1},x_{2},x_{3},x_{4})\in(0,1)^{4}, x_{1}+x_{2}<1, x_{3}+x_{4}<1\}$. Here is what I know about $F$: (i) $F(x_{1},x_{2},x_{3},x_{4})=0$ if $x_{1}=x_{3}$ and $x_{2}=x_{4}$ (self-similarity). (ii) $x...

Tagged differential-equations but mentions "partial". specify function given certain constraints
0
Q: Analysis and formal proofs.

YoTengoUnLCDEver since I started learning formal logic I've had these kind of doubts: Is analysis ever studied in a completely axiomatic/formal proofy way? What I mean is, given a set of axioms and inference rules, to prove things via formal proofs. Example: Let $f: \Bbb R\to \Bbb R: x\mapsto x$. Theor...

Short title. Tagged proof-theory. Analysis and formal proofs.
0
Q: Proving a homomorphism

icicleDefine $$\phi:\mathbb R[x,y]\rightarrow \mathbb R[x]$$ by setting $$\phi(P(x,y))=P(x,4)$$ Is $\phi$ a ring homomorphism? So I got the $\phi(a+b)=\phi(a)+\phi(b)$ part down, but am having trouble proving $\phi(ab)=\phi(a)\phi(b)$. How would I prove that without expanding out each term?

 
4:50 AM
-1
Q: Offensive flag declined

VVKI have marked http://stackoverflow.com/questions/34088758/how-to-make-android-studio-put-braces-on-own-line this post as offensive since it was containing offensive words. Why is my flag declined? The post is showing closed on my flag summary. When i visit the URL its unavailable. Why?

0
Q: На общей странице выборов нет StackOverflow на русском

Nick VolynkinЕсть общая страница выборов для Stack Exchange. У нас выборы начались, но сайта там нет. Raspberry Pi и Software Recommendations присутствовали там еще на этапе выдвижения кандидатов (как у нас сейчас). http://elections.stackexchange.com/

0
Q: Описание знака "Сторонник"

Ruslan GaripovВо-первых, не знаю почему знак "Student" переведён как "Сторонник". Возможно, это так и задумано. Но вот описание к этому знаку явно не окончено: На английском Stackoverflow это выглядит так:

 
0
Q: Prove that $\int_a^bxf(x)dx\geq\frac{b+a}{2}\int_a^bf(x)dx$

Kato yuLet $f:[a,b]\to\mathbb{R}$ be continuous and increasing, show that $$\int_a^bxf(x)dx\geq\frac{b+a}{2}\int_a^bf(x)dx$$ I am thinking of using integration by parts. First let $$F(x)=\int_a^xf(t)dt$$ Then $$\int_a^bxf(x)dx=bF(b)-\int_a^bF(x)dx=b\int_a^bf(x)dx-\int_a^b\int_a^xf(t)dtdx$$ So far I only...

0
Q: The function $f(r,\theta)=(rcos\theta,rsin\theta).$

neelaConsider the function $f:\mathbb{R}^{2}\rightarrow\mathbb{R}$ given by $$f(r,\theta)=(rcos\theta,rsin\theta)$$ I like to show that $f$ is one one in sone neighborhood of any non zero point $(r,\theta).$ I tried as $(rcos\theta,rsin\theta)=(scos\phi,ssin\phi)$ Which gives $r=s$ and $\theta=\phi+...

 
0
Q: Accepted answer + upvote were shown as +17 achievement

alecxeNot tagging as "bug" since I am not sure I can reproduce it again. Recently, my answer here was accepted and upvoted (while I was editing it). And I remember having no new achievement notifications. But, instead of expected +25, +17 showed up: Apologies if I'm misunderstanding something and...

 
Short title. Question contains please. The function $f(r,\theta)=(rcos\theta,rsin\theta).$
0
Q: describing the contor lines of constant x and y in the W plane of $f(z)=\frac{1}{z-1}$

Xfrglkvlet $f(z)= \frac{1}{z-1}$, Finding the real and imaginary parts $u=\frac{x-1}{(x-1)^2+y^2}$ and $v=\frac{-y}{(x-1)^2+y^2}$. I have to describe the contour lines of constant $x$ and $y$ in the W plane. I did something with python, not quite sure if I got it right, %matplotlib inline import matpl...

Questions tend to get more attention when they have a tag for a broad area of mathematics relevant to the question. Some of these tags might fit. (from a bot)Normal Human 21 secs ago
 
5:13 AM
0
Q: Challenging logic problem

NickUsing only modus ponens and substitution. Prove: q -> r -> [ [ p -> q ] -> [ p -> r ] ] using the three axioms: 1) p -> [ q -> p ] 2) s -> [ p-> q ] -> [ [s -> p] -> [ s -> q ] ] 3) p -> f -> f -> p where the symbol f is "false." I am having the hardest time trying to solve this proof, any ...

Welcome to Math.SE, Nick. Words such as challenging do not add information to titles. Please edit the title so that it better describes the specifics of your question. Do not hesitate to make it longer or include a formula if needed. More tips here. (autocomment)Normal Human 21 secs ago
0
Q: Which of the following metric spaces $X$ with the metric induced by usual Euclidean metric on $\Bbb R^n$ are complete?

Vikrant Desai A. $X=\Bbb Z \times \Bbb Z \subset \Bbb R \times \Bbb R$ B. $X=\Bbb Q \times \Bbb R \subset \Bbb R \times \Bbb R$ C. $X=(-\pi,\pi) \cap \Bbb Q \subset \Bbb R$ D. $X=[-\pi,\pi] \cap (\Bbb R - \Bbb Q) \subset \Bbb R$ I choose option A as an answer because $\Bbb Z \times \Bbb Z$ ...

0
Q: 3 variable cartesian plots?

offSpLso I have a problem. Say I had a variable n, which is equal to any real number between -10 and 10. than say I wanted to plot an x-y series based off each possible value of n. for example, say x = n, and y = n + 1. is there any ways that anyone knows of to make a single plot-able equation using ...

0
Q: Determine if the following expressions result in a scalar or vector field. If undefined, please explain why.

Neil PhilipF(x,y,z) is a vector field in space and f(x,y,z) is a scalar field in space. curl(grad(div(F))) curl(div(grad(f))) grad(div(curl(F))) grad(curl(div(F))) div(curl(grad(f))) div(grad(curl(f))) I'm trying to study for a multivariable final and I am having trouble understanding when and why these e...

0
Q: Isometry of the Poincare Half Plane

samsonite$D_{(0,s)}(x,y)=(sx, sy)$. Is this an isometry of the Poincare Half Plane and if so, how can this be proved?

Short question. Tagged proof-verification. Isometry of the Poincare Half Plane
 
5:29 AM
0
Q: Can i use SO account as a reference in my resume

un-luckyI have an account in SO, Can i use this account as a reference in my resume, to show my contributions? will it help me to get a position?

 
0
Q: complex reductive Lie group

XerxesI am reading A. L. Oniscik's paper Decompositions of Reductive Lie Groups, and the author cited a proposition that a complex reductive Lie group $G=ZS$ is locally isomorphic to the reductive algebraic group $\widetilde G= \mathbb C^{m}\times S$. I want to have a reference about this proposition....

0
Q: $A=\int _{|z+1|=2}\dfrac{z^2}{4-z^2}=?$

Amartya$A=\int _{|z+1|=2}\dfrac{z^2}{4-z^2}=?$ The function $f(z)=\dfrac{z^2}{4-z^2}$ has a pole on the points $z=-2,2$ which don't lie on the circle given by ${|z+1|=2}$ as $|-2+1|=1\neq 2$ and $|2+1|=3\neq 2$. Hence $f(z)$ is analytic on the given domain.Thus $A=0$. But answer is not matching .Ple...

Tall formulas in titles break the layout of question lists. Please replace \dfrac with \frac in the title. A title should not be all-MathJax; having some plain text helps with search and navigation. (from a bot)Normal Human 21 secs ago
0
Q: Basic probability.. has both a1 and a2 but not a3.

Talen KylonA system can have three different types of defects: A1, A2, A3. We are given the following probabilities. P(A1) = .12 P(A2) = .07 P(A3) = .05 P(A1 U A2) = .13 P(A1 U A3) = .14 P(A2 U A3) = .10 P(A1 and A2 and A3) = .01 What is the probability that the system has both type 1 and type 2...

 
0
Q: Is SO an 'official' Azure support channel

Michael BI asked / deleted a question yesterday that brought up an issue that I was curious about the answer to. On the Microsoft Azure Support page it lists SO as a support channel. Is this officially recognised by SO? or is it just a unilateral decision by Microsoft to send support cases here?

 
0
Q: Joint probability density function

user298814please help me with below Find the constant k so that F(x,y)={k(x+1)e-y , 00 Is a joint probability density function. Are X and Y independent ? Thanks in advance

Short question. Question contains please. Joint probability density function
0
Q: Solving a system of partial differential equations consist of 6 equations on 9 variables by using Matlab

farrokhI have a system of partial differential equations consist of 6 equations on 9 variables. $p = \frac{\partial f[x_1, x_2, x_3, y_1, y_2, y_3, z_1, z_2, z_3]}{\partial x_1};$ $q = \frac{\partial f[x_1, x_2, x_3, y_1, y_2, y_3, z_1, z_2, z_3]}{\partial x_2};$ $r = \frac{\partial f[x_1, x_2, x_3, y...

Tagged pde, differential-equations. Tagged differential-equations but mentions "partial". Solving a system of partial differential equations consist of 6 equations on 9 variables by using Matlab
0
Q: How this vector spans R^3?

user298815I am having trouble to understand why this given vector spans R^3. Vector={(−4, −6, 0), (2, −1, −4)} I understand that a non-zero determinant shows that the given set spans the space. However, this is not the square matrix. Therefore, I can not use determinant. I searched online and found this...

 
5:58 AM
posted on December 15, 2015 by Krom Stern

Предлагаю закрыть вот эти вопросы (и аналогичные им). IDE для языка Java Простой IDE для JavaScript IDE для C++ IDE для Python Списки IDE отлично гуглятся за 5 сек и даже представлены на ...

 
0
Q: Solve the equation:2^2x+1=(1/32)^x

NickHaving trouble with this problem: 2^2x+1=(1/32)^x Do I need to set the exponents equal to each other?

Short title. Short question. Solve the equation:2^2x+1=(1/32)^x
0
Q: Integral of a square wave

LesterberneHello guys i have a small question :) Why is the integral of a square wave with 50% duty cycle, centered at 0 not equal to 0? I mean a sine wave with these properties have an integral of 0. So why is the square wave's integral the area under the wave over half a period? Thanks a lot.

0
Q: State the midpoint rule

ModriscoState the Midpoint rule for: $$\int_c^dg(t)dt$$ with $m$ subintervals Is this how you state it? $$M_n=\frac{d-c}{m}[g(t_m)]$$ or $$M_n=\frac{d-c}{m}[g(t_1)+g(t_2)...g(t_m)]$$

 
6:24 AM
0
Q: Can any function be written as a composition of other functions?

Niki MCan any function be written as a composition of other functions? For example, can a polynomial h(x) be written as k(g(x)) where g(x)=x^2 and k is a function in the set of polynomials?

Consider adding a tag for a broader subject area to which the question belongs. Some of these tags might fit. (autocomment)Normal Human 21 secs ago
 
0
Q: How do I earn the Batman hat?

Vaulstein I recently saw that the User Community had a Batman hat awarded to it. Can a normal user be awarded a Batman hat? If yes, how?

 
0
Q: Complex Numbers (Locus)

dydxxThis is the question ==> (http://puu.sh/lBsGQ/0f8de6aea6.png) So what I have done is let z = x+iy (z-2)/(z+5) --> (x+iy-2)/(x+iy+5) --> [((x-2)+i(y))/((x+5)+i(y))] * [((x+5)-i(y))/((x+5)-i(y))] (rationalizing here) --> [(x-2)(x+5)+y^(2)+i(y)(x+5)-i(y)(x-2)]/[((x+5)^(2)+y^(2))] As arg((z...

Welcome to Math.SE, dydxx. This site uses MathJax formatting of formulas. More tips here. (autocomment)Normal Human 21 secs ago
 
0
Q: Should statement-of-purpose tag be synonyms with personal-statement?

The HiaryCurrently there are two tags that have the same function, however they are not set as synonyms: statement-of-purpose personal-statement Should we re-tag the "personal-statement" questions (only 7 questions) with the "statement-of-purpose" tag (177 questions). Or should someone with a higher ...

0
Q: Are my own answers counted for Tenacious and Unsung Hero badges?

KrumiaThe description for Tenacious, and Unsung Hero badges are: Tenacious: Zero score accepted answers: more than 5 and 20% of total. Unsung Hero: Zero score accepted answers: more than 10 and 25% of total. These descriptions does not explicitly say whether accepted answers that are s...

 
6:39 AM
0
Q: Same pattern of head and tail from the beginning or the end.

MathematicsA fair coin is tossed 8 times then find the probability that resulting sequence of heads and tails looks the same when viewed from beginning or from the end? How to approach this question because making cases would be such a difficult task?

 
7:00 AM
0
Q: Please help with a general solution of a functional equation involving projections

user98231I saw the following claimed : Let's say we have the functional equation $f(R+S) = f(R) + f(S)$ where R and S are projections in a vector space, and f is a real valued function. Then its general solution is : $f(R) = c\ Tr(R)$, where $c$ is some constant and Tr is the trace. The reason given ...

 
0
Q: Can dupe hammer power be abused to obtain the I'm Batman hat?

EranThe scenario can be quite simple: Find a question that was closed as duplicate (that I did not participate in closing) having a tag for which I have a gold badge. Make some edit to that question. Use my binding vote to reopen that question. This seems to fit the hat description perfectly - ed...

0
Q: Tab Tab Tab, is this browser on

HennesWe currently have multiple tab tags open on our site: tabs Tabs are a graphical user interface feature that allows multiple documents to be contained within a single window. browser-tabs The tag has no usage guidance, can you help us create it? The last possibly from an earlier cleanup. I ...

 
0
Q: Algebra Roots (Cubic/Complex)

dydxxThis is the question --> http://puu.sh/lW9XL/b1e04691d2.png I've dealt with quadratics in this form but never with cubics. I tried searching up the cubic formula but I think it would take way too long to be practical in an exam situation..

0
Q: Check differentiability at x = pi/2

Parag GuptaLet f:R->R be the function degined by f(x) = Sin[x]/(|x|+Cos[x]) check differentiability at x = pi/2

Title ends with a digit. Short question. Check differentiability at x = pi/2
0
Q: Finding a harmonic function,

User001The problem statement is: Suppose that the real series $∑_0^{∞} a_n$ and $∑_0^{∞} b_n$ converge absolutely. Part 1 Prove that there is a function $u(r,θ)$ which is harmonic in $1<r<2$ and continuous onto the boundary such that $$u(1,θ)=∑ancos(nθ)$$ and $$u(2,θ)=∑bncos(nθ)$$ Part 2 Is $u(r,...

0
Q: Let $f(x)=\frac{\alpha x}{x+1},x\neq-1.$Then for what value of $\alpha$ is $f(f(x))=x?$

Vinod Kumar PuniaLet $f(x)=\frac{\alpha x}{x+1},x\neq-1.$Then for what value of $\alpha$ is $f(f(x))=x?$ Given $f(f(x))=x$ $\Rightarrow f(x)=f^{-1}(x)$ means we need to find the point where the function and its inverse intersect and such points are found on the line $y=x$ so we need to solve $f(x)=f^{-1}(x)=x...

 
7:21 AM
0
Q: Discussions on edit review

Ja8zyjitsRecently I made an accepted edit and the summary that I wrote was not verbose enough or descriptive. All I intended to do was to improve the readability of the question as the user was new and dint knew how to frame a question properly or edit with proper codes, he wrote his test code in comments...

 
Welcome to Math.SE, Michael. This site uses MathJax formatting of formulas. More tips here. (from a bot)Normal Human 20 secs ago
0
Q: General formula to compute the exponent of the symmetric group $S_n$

Omar ShehabSomeone has already asked whether an exponent less than $n!$ is possible for a symmetric group $S_n$. It has been answered that it is for $n \ge 4$. I would like to know if there is a general formula to determine the exponent of $S_n$.

 
0
Q: I see not wrapping comments on Firefox 42.0

Soner GönülI see not wrapping comments on Firefox 42.0. Take a look at this answer's comment. There is no problem on IE 11 and Google Chrome 47.0.2526.80m versions. I search on MSO as comment wrap and only related question I found is Messy SO interface on Firefox which is tagged as status-completed. And ...

-1
Q: Recover to SO account

Mithlesh UpadhyayMy this question is linked with that question : What should I do for my these questions? [duplicate] I'm guessing that this may not be duplicate. Please, my intention is not get either upset or hurt anyone. I'm respecting you all and make to welcome you. My this question : Output of printf (“C...

 
7:49 AM
0
Q: Measure of sums of sets

MathmathIs it true that $$m(A+B)\geq m(A)+m(B)$$? For any measurable sets $A,B$ where $A+B=\{x+y,x\in A,\,y\in B\}$ Thanks

Short title. Short question. Measure of sums of sets
 
8:11 AM
0
Q: Solve $[x]=m\{nx\}$

User 1upon0Find the sum of all solutions of the equation $$[x]=m\{nx\}$$ where $m,n$ are positive integers. I don't know, it is just making a lot of cases. Give some hint. Thanks.

Short title. Short question. [Solve $[x]=m\{nx\}$](math.stackexchange.com/q/1576447)
 
0
Q: R Text naive bayes gives different classification

ssetWe are using R naive bayes for text classification. Results are different from hand calculated. Maybe R is performing some normalization, distribution and does not work in multinomial mode (text words with frequency). Also I am unable to understand how does R naive bayes compute prior conditional...

 
0
Q: Convergent of infinite series

kalpeshmpopatThe sum of the series $1+\frac{1+2}{1!}+\frac{1+2+3}{3!}+....$ equals? The answer is $\frac{3e}{2}$. But I dont Know How? I have tried following: $1+\frac{1+2}{1!}+\frac{1+2+3}{3!}+....$=$\sum\limits_{n=1}^{\infty} \frac{n(n+1)}{2n!} $ I know that sequence of partial sum $S_n$=$1+\frac{1+2}{...

0
Q: Cesaro summation and convergence

LeonI am trying to makes sense of the proof to following problem: Given: $A_n = \displaystyle \frac{\sum_{k=1}^n a_k}{n}$. Can $\{A_n\}$ converge if $\{a_n\}$ diverges; $\forall n,a_n>0; \limsup{a_n}=\infty$. Proof: Lets define $a_n = \begin{cases} k & n=k^3\\ \frac{1}{k} & n \in (k^3,(k+1)^3) \...

Please don't use (self-learning) tag just because you were self-studying. This tag is only for questions about the process of self-studying. (autocomment)Normal Human 26 secs ago
0
Q: Integral of exponential

MathematicingI have the expected value of the square of the position operator given as $$\langle x^{2} \rangle= \int_{-\infty}^{\infty} x^{2}e^{\frac{amx^{2}}{h}}dx$$ I understand that the integrand can only be evaluated by the form $$ \int_{-\infty}^{\infty} x^{2} e^{-ax^{2}}dx=\frac{1}{2}\sqrt{\frac{\pi}...

 
8:37 AM
0
Q: Body content filter rejects posts with Chinese characters, but with a wrong error message

200_successAfter migrating a question to Stack Overflow (Using a covering index to select records for a certain day), I attempted to make some edits, but found I could not save the text. Specifically, the error was: Of course, that sounds like a silly content restriction. It turns out that no matter wh...

 
0
Q: Compute unexplained variance in bayesian updating problem

BreugemSuppose there is an unobserved RV that can take two levels only (i.e., V~Bernoulli with values {0,1} and a 50%/50% probability). In my model there is a "signal" Y available that allows one to learn about V. Without focusing on the learning problem, lets just simply assume that upon the observati...

This site uses MathJax formatting of formulas. Consider adding a tag for a broader subject area to which the question belongs. Some of these tags might fit. More tips here. (from a bot)Normal Human 25 secs ago
 
8:50 AM
0
Q: Solve a matrix equation..

user25778I am at moment trying to solve a system of linear equation, and I am not sure if the value I retrieve is even possible, or my program return some garbage value... The equation i am trying to solve is : J(q)dq=du J(q) = [6x7] matrix dq = (not known) du = [6x1] So since i have to solve for dq ...

0
Q: Trouble in understanding Sigma-algebra definition

user298844I have trouble in understanding the definition of $\sigma $ - Algebra. I tried googling but couldn't make much of it.

Title contains troubl. Short question. Trouble in understanding Sigma-algebra definition
0
Q: Parabola conic section tangents

SamagraFind the equations of the tangents of the parabola y^2 = 12x, which passes through the point (2,5). I tried to do SS1=T^2 But from that i am not getting Please can you do it for me

Short question. Question contains please. Parabola conic section tangents
 
1
Q: Synonyms: jdk9 == java-9

assyliasI suggest doing the same as for Java 7, 8 etc.: main tag: java-9 (17 questions) synonym: jdk9 (3 questions)

-3
Q: How do I earn the Odinson hat?

eckesIt's description says it's a secret hat. How do you earn it?

0
Q: Question should be reopen?

ketanToday i have reviewing the reopen vote. Where i show one question . Which is mark as duplicate. OP edit this question with new question. My next question is that blah blah blah... Should i reopen this new question or leave is closed? Can we ask related new question if it is mark as duplicate?

 
9:28 AM
0
Q: Divisibility of $2^n-n^2$ by 7

billybobHow many positive integers $n<10^4$ are there such that $2^n - n^2$ is divisible by 7?

0
Q: Suppose $f$ and $g$ are holomorphic in $G$ and $\gamma \sim_G0$. Prove that if $f(z)=g(z)$ for $z\in\gamma$, then $f(z)=g(z)$ for $z$ in $\gamma$.

LehtiaSuppose $f$ and $g$ are holomorphic in the region $G$ and $\gamma \sim$ $_G$ $0$. Prove that if $f(z)=g(z)$ for all $z\in\gamma$, then $f(z)=g(z)$ for all $z$ inside $\gamma$.

0
Q: Calculate finite p-series

prometheus21Let $\sum_{k=a}^{n} \frac{1}{k^{1/2}}$ I have written a simple C++ program that computes the series for different values of n. What is the mathematical approach to finding this series.

Short title. Short question. Calculate finite p-series
0
Q: A boat is traveling at 30 knots/hour at 30 degrees E of N. Find the components of the velocity vector

JK3Would it be 15 knots per hour or like 15sqrt(3) knots?? I'm not sure if I understand this please help.

 
0
Q: December double site competition around hats!

Mark MayoFor those competitive, please simply edit the two answers below when you want to 'enter' the competition. It'll simply be: whoever gets the most combined numbers of hats on Travel.SE and on Expats.SE, in an effort to help both sites. That is, if you have 7 on travel and 3 on expats, your total ...

-1
Q: send files who starts with charachter "android"

 redai want to send files from /mnt/sdcard into another device via bluetooth.... i already figure out how to do that but the problem is that i want to send the files that starts with specific charachter "FichierFinale" cause in my direcory i have "FichierFinale1,FichierFinale2....." so i have to ...

 
9:44 AM
0
Q: [LINE SEGMENTS]

tutifreshThere is a group of 30 line segments of different length such that a triangle can be formed by each three of them. Find the maximum number of different obtuse triangles formed by segments from that group.

Short title. [[LINE SEGMENTS]](math.stackexchange.com/q/1576514)
0
Q: Finding the points on the graph of a function

Ampplease help me. Please. Find the points on the graph of ⅓ x3+x2-x-1 at which the slope is (a)-1; (b)2 I don't know where to begin.

Short question. Question contains please, please. Finding the points on the graph of a function
 
0
Q: Анонс Winter Bash 2015

Nicolas ChabanovskyВот и еще один календарный год подходит к концу, а это может означать лишь одно. Снова наступает время события, которое дарит всем удовольствие – «Winter Bash»! Начиная с сегодняшнего дня и до конца 3-го января 2016 года вы сможете зарабатывать шляпы за различные достижения на Stack Exchange и...

0
Q: Dupehammering widely-viewed positively-received Qs with lots of up-votes towards rarely-viewed Qs?

IkeThere is a recent question I've favorited (starred) and been monitoring which has a lot of positive attention, views, and up-votes. The question has dozens of up-votes along with the answers, and there's a very interesting dynamic brewing there. It is not a question I'm participating in directly...

 
0
Q: Show that if $a>1$ then $\log a - \int_a^{a+1} \log x dx$ differs from $\frac{-1}{2a}$ by less than $\frac{1}{6a^2}$

luka5z Show that if $a>1$ then $\log a - \int_a^{a+1} \log x dx$ differs from $\frac{-1}{2a}$ by less than $\frac{1}{6a^2}$. For some $\theta$ between $a-1$ and $a$ and odd $n\in\mathbb{N}$ we have equality: $$\log a - \int_a^{a+1} \log x dx=\log a - \int_{a-1}^atdt+\int_{a-1}^a \frac{t^2}{2}dt-...

 
0
Q: In the new tabbed navigation what happened to "Load New Questions"?

John CWhen tabbed browsing was first brought in as new questions came in a small message would appear below the filter detailing the quantity of questions and a "Load New Questions" button next to it. The number of new questions appears next to the tab label but the message and button no-longer appear...

 
10:08 AM
0
Q: $\left|\left| f \right|\right|_{BV} = \left| f(a) \right| + V_{a}^{b} f$ defines a norm in the space $BV[a,b]$

Von KarI'm learning about functions of bounded variations and need help to solve this problem: Show that $\left|\left| f \right|\right|_{BV} = \left| f(a) \right| + V_{a}^{b} f$ defines a norm in the space $BV[a,b]$. My attempt and Thoughts: First. We want to show that $\left|\left| f \right|\ri...

Tagged proof-verification. [$\left|\left| f \right|\right|_{BV} = \left| f(a) \right| + V_{a}^{b} f$ defines a norm in the space $BV[a,b]$](math.stackexchange.com/q/1576540)
 
0
Q: В чем интерес быть модератором?

Andrey K.Почему некоторые участники стремятся стать модераторами? В чем здесь интерес? Что ими движит? Как по мне, то бывало, что расстраивался, что потерял какой-нибудь час на СО, когда мог бы потратить его на более насущные вещи. Да, конечно, пользы много, когда чего-то не знаешь и тебе подсказывают. Н...

1
Q: Editing: am I just doing it wrong?

TwoStrawsI'm trying to contribute by editing posts where code is formatted badly, the English is poor, or both. I've had a couple of edits rejected under the "doesn't make it even a little bit easier to read" reason, which I found dubious enough, but I just had a reviewer propose rejection for "this edit ...

0
Q: The use of "trig" as an abbreviation of "trigonometry".

gebruikerEvery now and then you see the word trigonometry being abbreviated as trig. Now the meanig of trig is usually quite clear from context, however I'm having doubts on how common/accepted this is. I especially wonder how this affects seachability through Google (or any other search engine), sinc...

 
0
Q: Find a limit of a set

NoamI got this set - $lim_{ n _{\to \infty}} \left(\left(\sqrt{9x+1}-\sqrt{3x}-\sqrt[3]{2x}\right)\left(\sqrt{2x+1}\right)\right)$ I know that $lim_{ n _{\to \infty}} = \infty$ by intution. But I dont know how to calculate that. Thanks a lot.

0
Q: What are the prime ideals in $\frac{\mathbb{Q}}{\langle x^n-1 \rangle}$

Aloysius GodinhoWhat are the prime ideals in $\frac{\mathbb{Q}[x]}{\langle x^n-1 \rangle}$ for $n=4$ and $n=5$? How do I tackle this?

0
Q: Solving a solved Geometric problem in an alternate way.

MickThis question (circles, power of point, cross ratios ) has been solved using Steward’s theorem. I try to solve it in another way but find myself stuck at some point. If we let $BC = x, CD = y, DP = z$, the required equality ($CD \cdot BP = BC \cdot DP$) is just $y(x + y + z) = x(z)$ or more si...

0
Q: Find a field K where the vectors are linearly dependent in the vector space K^3

TearsdontfallsMy task is to find a field $K$ where the vectors $$ \begin{pmatrix} 2 \\ 3 \\ 1 \\ \end{pmatrix} , \begin{pmatrix} 0 \\ 4 \\ 0 \\ \end{pmatrix}, \begin{pmatrix} 0 \\ 1 \\ 2 \\ \end{pmatrix} ...

0
Q: Showing a power series converges...

CoffeeToTheoremsI'd like to show that $$\sum\limits_{n = 1}^\infty {{{{x^{n + 1}}} \over {n(n + 1)}}} $$ absolutely converges for |x| < 1

0
Q: Complemenet of Matching graph

FrankI'm trying to understand whether the complement of a Matching is an Edge-cover and, in that case, whether the rule is valid for the opposite situation. By doing some examples I see that the complement of a Matching seems to be always an Edge-cover (is it correct?) but the complement of an Edge-c...

 
10:48 AM
0
Q: derivative of lebesgue integrable function

chaviaras michalisSuppose we have $f \in L(I)$ and derivative $f'$ exists almost everywhere . It is $f'$ measurable ? I have no idea how to begin to construct the proof .

0
Q: Prove that $\mathbb{Z}_p\oplus\mathbb{Z}_p\oplus\mathbb{Z}_p$ is not generated by 2 elements

almogI need to prove that the group $\mathbb{Z}_p\oplus\mathbb{Z}_p\oplus\mathbb{Z}_p$ is not generated by 2 elements. How?

0
Q: A property of Poisson process

andreasvrLet $Y_t$ be a centered Poisson process, why \begin{equation} \lim_{n \to \infty} \sup_{s<t} |n^{-1}Y(ns)| = 0 \qquad a.s. \end{equation} This is a fundamental step in the proof of the law of large number for continuous time Markov Chain. I'm following the proof on the book by Ethier and Kurtz b...

0
Q: Finding best linear unbiased estimates

user46697Is sample mean always the BLUE of population mean?

0
Q: How is the SAGE Mathematical system tested -- is there a standard data set of problems

user1172468To test the correctness of the SAGE software, how is it tested? Is there a standard set of math problems that SAGE is asked to solve as part of the build process?

Questions tend to get more attention when they have a tag for a broad area of mathematics relevant to the question. Some of these tags might fit. (autocomment)Normal Human 21 secs ago
0
Q: Injection and Bijection of the function $f(x,y)=(\frac{x}{1+x+y},\frac{y}{1+x+y}).$

neelaLet $A=\{(x,y)\in\mathbb{R}^{2}:x+y\neq-1 \}$ Define $$f:A\rightarrow\mathbb{R}^{2}$$ by $$f(x,y)=(\frac{x}{1+x+y},\frac{y}{1+x+y}).$$ I have to prove that the the function $f$ is one one but not onto. It is clear that $f$ is continuously differentiable but Inverse map theorem gives locally inje...

 
11:03 AM
0
Q: deleting junk spam more quickly

WolfgangIn the last days, there have been several posts which are commercial spam (mostly from India, as it seems) and have no link at all with mathematics. Is it technically possible that one of the moderators just deletes such posts as soon as a "flag as spam" comes in, without putting it on the revisi...

 
0
Q: Time scaling in Laplace transformation

John RailmanLaplace transform property: $f(at)\leftrightarrow \frac{1}{a}F(\frac{s}{a})$ where $f\leftrightarrow F$. Question: Is $a>0$ necessary for this property?

0
Q: Let $S$ be any set of statements. How do I concisely show that $\sim$ is reflexive, symmetric, and transitive on $S$?

crocketThe following LaTeX document renders exercise 2.5.2 from "Mathematical Logic" by Ian Chiswell, Wilfrid Hodges, Oxford University Press (2007). \documentclass[oneside,12pt]{book} \usepackage[a4paper]{geometry} \usepackage{microtype} \usepackage[T1]{fontenc} \usepackage{enumitem} \usepackage{amsm...

0
Q: Reference for a Heat Process in a Wedge

Boggie GeorgievI would like to ask about an explicit suggestion/reference for the following type of heat processes: Roughly, assume we have a "wedge" $W$ of the following form - a domain in $\mathbb{R}^n$ with a tip at 0, having some hyperplanes (passing through 0) as sides and, say, the piece of the unit cyll...

Tagged pde, differential-equations. Tagged differential-equations but mentions "partial". Reference for a Heat Process in a Wedge
0
Q: On the sum of digits of $f(n)$

alex alexeqThis is probably wel known, so please let me know. Let $s(n)$ be the sum of all digits of the integer $n$ (in base 10). Consider a polynomial $f$ with integer coefficients. I want to show that for any large enough $n$, $s(f(n)) \leq s(n!)$. Any suggestion would be helpful.

Question contains please. On the sum of digits of $f(n)$
0
Q: Help with understending the exponential integral.

user293846Define the exponential integral $E(z)=\int_{-\infty}^z\frac{e^t}{t}dt$ (this differs from the usual definition of $Ei(z)$ only by $\pm i\pi$). Consider now the following contour integral of a function $f(z)=\frac{e^{az}}{z-b}$: the contour starts from real $-\infty$, goes to 0, then to imaginary...

Words such as help are uninformative in titles. Please edit the title so that it better describes the specifics of your question. Do not hesitate to make it longer or include a formula if needed. More tips here. (autocomment)Normal Human 23 secs ago
 
11:21 AM
0
Q: How can I spot the Winterbash related mischief going on?

Quill - HAT MANIACI've noticed some strange behaviour going on since Winterbash has started, how can I spot this behaviour before it turns into a serious issue? How is this behaviour related to hats?

 
0
Q: find the FCP X/Y

KYRIAKOSSuppose , X Y independent geometric random variables with the same parameter p.we want to find the captive probability function of X given that X+ Y =n, where n >1.

Short title. Short question. find the FCP X/Y
0
Q: Constructing a Riemann integral...

CoffeeToTheoremsI want to evaluate the Riemann integral $\int_0^1 {{x^2}dx} $ I want to find upper and lower estimates of the form: $$U \ge {1 \over {6{N^3}}}(N(N + 1)(2N + 1))$$ $$L \ge {1 \over {6{N^3}}}(N(N - 1)(2N - 1))$$ Then show they're equal and then evaluate the mentioned Riemann integral. I think the ...

Consider adding a tag for a broader subject area to which the question belongs. Some of these tags might fit. (autocomment)Normal Human 22 secs ago
 
-1
Q: A 404 error occurred Page not found. The requested URL could not be matched by routing. No Exception available

AdiI have implemented the stripe system on my website using stripe documentation https://stripe.com/docs/tutorials/checkout I have made a payment file and a charge file . this is the code for payment file <div class="panel-body"> <form action="in/payment/charge.phtml" method="POST"

 
0
Q: What does it mean if a free algebra has an unsolvable word problem?

Thomas KlimpelI wonder how hard identity testing (similar to polynomial identity testing) can be for a free algebra. I thought that in a certain sense, the problem should always be semi-decidable, because the free algebra is defined with respect to the identities which follow from the given set of equational a...

0
Q: Distributing arbitrary union over arbitrary intersection (and vice versa)

ArcindeSuppose $I, J$ are arbitrary index sets. Let $X$ be a set and $X_{i, j} \subseteq X$. I claim that $$\bigcap_{i\in I} \bigcup_{j\in J} X_{i,j} = \bigcup_{(j_i) \in J^I}\: \bigcap_{i\in I} X_{i, j_i}.$$ $$\bigcup_{i\in I} \bigcap_{j\in J} X_{i,j} = \bigcap_{(j_i) \in J^I}\: \bigcup_{i\in I} X_{i,...

0
Q: polycyclic group

user182085Let $H$ be a subgroup of a finitely generated polycyclic group $Q$ such that intersection of $H$ with each normal subgroup of $Q$ is non trivial. I need an example to show that it is not necessarily that $H$ is normal in $Q$.

Short title. polycyclic group
 
11:57 AM
0
Q: How can I see where I have used myopenid as login?

bluebrainSome time ago I was using myopenid.com on SO sites. Traveling around, I have seen it is still registered on one of sites or settings (within a title something alt openid or alt login etc.). Unfortunately I lost it suddenly. I want to remove it because I do not use it anymore. Where can I find it,...

 
0
Q: Lagrange interpolation uniqueness

NesaHow do I show that Lagrange's polynomial is the only one (with degree < n) that takes the given values at given points? ($f(x_{1})=y_{1} \space f(x_{n})=y_{n}$)

Short title. Short question. Lagrange interpolation uniqueness
0
Q: minimum number of straight lines needed to cover n points

MehrdadSuppose we are given a set of n points in the euclidean plane , they are distributed arbitarily ( not in general position). what is the minimum number of lines in the plane needed to cover them all?

Welcome to Math.SE, Mehrdad. Questions tend to get more attention when they have a tag for a broad area of mathematics relevant to the question. Some of these tags might fit. (autocomment)Normal Human 21 secs ago
0
Q: Compare Valuations

user261123Let $K$ be a field complete with respect to a discrete valuation $v$. Let $K_s$ its separable closure, $w$ the unique valuation extending $v$, $(\mathcal{O}_{K_s}$, $\mathfrak{M})$ respectively the ring of integers and its maximal ideal of $K_s$. Let $p \in \mathbb{Z}$ be a prime number such th...

Short title. Compare Valuations
0
Q: Proving that minimal blocks have a vertex of degree 2

Clara McCarthyI'm trying to show that if we have a graph G that is a minimal block with at least 4 vertices, that one the vertices must have degree 2. We have defined a minimal block as a 2-connected graph such that that the removal of any edge e results in a subgraph G-e that is not 2 connected. I'm assumin...

0
Q: How to solve y'=x^2+y^2, y(0)=1

SergeiHow to solve y'=x^2+y^2; y(0)=1; Specify any segment in which there is a solution of this equation and to construct successive approximations y0, y1, y2 to the solution of this equation. Help me please.

Question contains please. How to solve y'=x^2+y^2, y(0)=1
0
Q: How do I prove that $lim \sqrt(x)=3$ while x->9 using epsilon delta proff

jaldkHow do I prove that $$lim \sqrt(x)=3$$ while x->9 using epsilon delta proff

 
12:35 PM
0
Q: Why did my question still get marked as duplicate even after my clarification?

uhohI asked this question "How to recover a pickled class and it's instances" and after I accepted an answer, it was marked as a possible duplicate. I added a substantial comment to the question explaining why this question is different than the one flagged by the duplicate message. note: the pickl...

 
12:59 PM
0
Q: cardinals arithmetic prove that k2^m>=k1^m if k2>=k1

Tomplease I need help. k1, k2, m powers. given that k2>=k1 prove that k2^m>=k1^m. I know that I need to find a one to one function f from k1^m to k2^m in order to prove that. I also know that if |A|=k1, |B|=k2, |C|=m and k2>=k1 there is a one to one function g from A to B. Is someone can guide m...

0
Q: What are Fuzzy integrals?

Angel BlascoHow could you define "Fuzzy Integral"? Could you recomended me any interesting book to self study about Fuzzy integrals? Thanks

Short title. Short question. What are Fuzzy integrals?
0
Q: properties of outer measure

Al jabraLet X be a set and an outer measure on X is a function $\theta:\mathcal{P} X \to [0,\infty]$ such that (i) $\theta \phi= 0$ (ii) if $A \subseteq B \subseteq X$ then $\theta A \leq \theta B$. (iii) for every sequence $<A_n>_{n \in \mathbb{N}}$ of subsets of X, $\theta(\cup_{n \in \mathbb{N}} ...

0
Q: Maximal ideals of $D$.

Alex Cg.Let $D=\{f:\mathbb{R}\rightarrow \mathbb{R}\,|\, $f$ \,\mbox{ is twice differentiable}\,\}$ be a ring and let $J=\{f\in B\,|\, f'(0)=f(0)=0\}$ be an ideal in B. Find all maximal ideals of $B$ that contain to $J$. I prove that the sets $M_1=\{f\in B\,|\, f(0)=0\}$ and $M_2=\{f\in B\,|\, f'(0)=0\}...

0
Q: COV(Y,Z),VAR(X)>=VAR(Y)+VAR(Z)

KYRIAKOSLet X=Y-Z, where , YZ is a non- negative random variables for which applies YZ = 0. a) Show that cov (Y,Z) <=0 b ) Show that var (X)>= var (Y) + var (Z)

Short title. Short question. COV(Y,Z),VAR(X)>=VAR(Y)+VAR(Z)
 
0
Q: How low quality does an answer have to be to be recognized as low quality

vikingosegundoFew days ago I flagged an answer as low quality, that answered the question how to dadd several date to a NSDate object in swift by saying you should create an extension, but leaves the task of explaining this to a link, that is dead ever since. Also it does also not mention that for the shown li...

 
1:17 PM
0
Q: $X$ and $Y$ are two sets and $f:X\to Y$.If $\left\{f(c)=y;c\subset X,y\subset Y\right\}$ and $\left\{f^{-1}(d)=x;d\subset Y,x\subset X\right\}$

Vinod Kumar Punia$X$ and $Y$ are two sets and $f:X\to Y$.If $\left\{f(c)=y;c\subset X,y\subset Y\right\}$ and $\left\{f^{-1}(d)=x;d\subset Y,x\subset X\right\}$ then the true statement is $(A)f(f^{-1}(b))=b\hspace{1cm}(B)f^{-1}(f(a))=a\hspace{1cm}(C)f(f^{-1}(b))=b,b\subset y\hspace{1cm}(D)f^{-1}(f(a))=a,a\subset...

0
Q: Prove that $K$ is algebraically closed

Alex Cg.Let $K$ be a field of characteristic zero and $p$ a prime number such that $p^2$ divide at degree of all irreducible polynomial not linear in $K[x]$. Prove that $K$ is algebraically closed.

0
Q: Given a polynomial $p$, then $\forall K>0$, $\exists r_k$ such that $|t|\geq r_k \Rightarrow |p(t)|\geq K$

SantosLet $p: \mathbb{R} \rightarrow \mathbb{R}$ be polynomial $p(t) = a_0 + a_1 t+ \cdots + a_n t^n $ $(a_n \neq 0)$. I'd like to prove the following statement by induction: $\forall K>0$ there exists $r_k$ such that $|t|\geq r_k \Rightarrow |p(t)|\geq K$. My attempt Base case: For $n=1$, we have ...

0
Q: Why does every ring has non-zero divisor

Alex ChanCan there be a ring such that all the elements are zero divisor? As I read about the poof of every finite commutative ring has unity, the prove is to use one of the non zero divisor namely 'a' as the permutation. So if a^0b,a^1b ... a^nb=a^0b gives a complete cycle on every non zero element b, a^...

Welcome to Math.SE, Alex Chan. This site uses MathJax formatting of formulas. More tips here. (autocomment)Normal Human 21 secs ago
0
Q: Is there a nice way to tell when two transfer-matrices produce the same recurrence relations?

Gadi AIn Combinatorics, transfer matrix is a matrix defining linear recurrence relations. I am asking specifically about the matrix $\left(\begin{array}{cccc} 2 & 1 & 0 & 0\\ 1 & 1 & 0 & 0\\ 0 & 0 & 2 & 0\\ 1 & 0 & 0 & 2 \end{array}\right)$ Which defines the following set of recurrence relations: $a...

 
1:32 PM
-4
Q: "Everybody gets a hat" idea

Kengyou know, the "Everybody gets a hat" makes a much better mustache than a hat...how about a Movember event or at least some beard and mustache options 80{)>

 
0
Q: In Z/(12) modulo 12 ring the LCM of (6,8) Does Not Exist ? HOW?

TehseenIn Z/(12) modulo 12 ring the LCM of (6,8) Does Not Exist ? HOW ?

0
Q: trigonometric equation using compound angle formulas

ryanmcIs it possible to evaluate these 2 calculations using a compound angle formulae and not a calculator? Sin(105°) Cos( - 30°)

 
0
Q: Community bulletin shows regular post in featured post section

Aziz ShaikhCommunity bulletin is having Moderator Election Results in the Featured on Meta section, even though that post does not have featured tag on it anymore.

0
Q: Why does an open bounty make a question unclosable?

brhansThis question has attracted at least 7 down-votes (1 mine) and no up-votes and in my opinion seems overly broad and at least a bit off-topic. Comments from other users have indicated as much too. But it can't be closed because it has an open bounty which only expires in 6 days... Surely having ...

 
1:50 PM
0
Q: Orthogonal Basis Gram-Schmidt process

BobHow would I go about solving this? Before you ask, I don't even know where to start :(

0
Q: Inverse LaPlace transform of complicated function

PhoneheadsI have a LaPlace transformed function that I'd like to transform back. It's quite a complex function however, which is why I am stuck C(x,s) = $(\frac{m}{s})$*e$^{\lambda^2*(\frac{v-\sqrt{\frac{4*D*s}{\lambda^2} - v^2}}{2*D})* x}$ I'd know the inverse LaPlace transform of a less complex but co...

 
2:10 PM
0
Q: Prove that if $A$ is not invertible, $\lambda=0$ is one of the eigenvalue.

CoolKidI only know how to prove the reverse. Anyone give any ideas for this proof?

0
Q: Colored beads on a loop

WoodyaSuppose we have $p$ beads of $n$ different colors on a loop. $p$ is a prime number and we consider the loop to be the same if one is a rotation of the other. Then how many distinct beads are there? By using Burnside's Lemma, I have the result of $\frac{(p-1)n+n^p}{p}$, but not quite sure about my...

0
Q: prove that $\lcm(a , b) = \prod_{n=1} (P_i)^\max(alpha_i, beta_i$

GeoffI tried expanding out on the left hand side using the formula for the LCM but i couldn't see where to go from there.

 
-5
Q: Stackoverflow general usage

Dim DIt is quite annoying the stackoverflow reputation and it is not clear how to improve it. I actually have a negative one that prevents me posting answers and leaving comments. Each time it says that my account is not allowed to do it and advices me to see the help center page. When I go on this pa...

-7
Q: Feature for poking a question to top

ElltzLike the way bounty works which gives special attention to question etc etc. also like the way the system works that occasionally push questions to the top etc etc. i was wondering if you guys could add a feature maybe like the way bounty works, you offer reps to push your question to the top o...

 
0
Q: Tangent Space to Grassmannian

slin0I have a second question today. In Harris' "Algebraic Geometry: A First course" he constructs (on page 200) an isomorphism between the tangent space of the Grassmannians and some homomorphisms: He begins with the cover of the Grassmannians defined in Section 6, i.e. for a (n-k)-plane $\Gamma \s...

0
Q: Is this OEIS assumption re the "congruent number problem" correct?

Tito Piezas IIII. Wikipedia: An integer $n$ is a congruent number if there are rationals $a,b,c$ such that, $$a^2+b^2 = c^2\\ \tfrac{1}{2}ab = n$$ or, alternatively, the elliptic curve, $$x^3-n^2x = y^2\tag1$$ is solvable in the rationals. II. Mathworld and OEIS: They define them (see this and this) furth...

0
Q: Lower semicontinuous in geometric measure theory

jwgohI am currently working on this problem in Frank Morgan's Geometric Measure Theory book. $\mathbf{M}$ is $\mathbf{F}$ lower semicontinuous on $\mathscr{D}_m$. What I have done so far: $T_i \to T$ under the real flat norm: $\forall \epsilon>0, \exists N$ such that $\min \{ \mathbf{M}(A-A_i) + \...

Questions tend to get more attention when they have a tag for a broad area of mathematics relevant to the question. Some of these tags might fit. (from a bot)Normal Human 20 secs ago
0
Q: sinusoidal functions

ryanmcI need to plot a graph of the following function for values time, 0≤t≤2π0t2π (radians). y=5sin(2t+π6) y5sin2tπ6 I need to describe and define the amplitude, periodic time and frequency. Can the formula be put into Excel somehow to plot the graph? Thanks trigonometry graphing-functions

Short title. sinusoidal functions
 
2:29 PM
-2
Q: Is there any feature to Vote To Accept

AbdullaIncident Some new users join to stack family. And they ask some thing hard or best question. So our Stack buddy's answer that question. And the answer was an very advanced one and Superb one.(as my knowledge). But what OP do is place an comment Its worked.... rather than accepting it. and thy n...

 
0
Q: boys and girls statistic problem

KYRIAKOSIn a class consisting of 18 boys and 12 girls. Professor asks a question . Each boy has a probability 1/3 to know the answer, while corresponding probability for each girl is 1/2. What is the expected number of students who can answer correctly ?

Title contains problem. boys and girls statistic problem
0
Q: How would you show that Ax=b has a solution $\iff$ rank A = rank [A b]?

1412I am wondering what the [A b] here stands for, is it matrix and its multiplied result? So could anybody please give me some hints to start the problem as I only have the intuition that if Ax=b and x is not $0$, then rank A has to be equal to [A b].

Question contains please. [How would you show that Ax=b has a solution $\iff$ rank A = rank [A b]?](math.stackexchange.com/q/1576833)
 
0
Q: Arduino question on Stack Overflow or Arduino Beta

Mike159Somehow I only recently realised there is a Arduino Stack Exchange site. In the past I would consider asking Arduino software questions on SO and anything that related to hardware issues (including software depending on/manipulating hardware) on Electronics Stack Exchange. Should all Arduino que...

 
0
Q: Confusion in evaluating the limit $\lim_{x\to-\infty}\sqrt{x^2+ax}-\sqrt{x^2+bx}$

diyaI was solving a question related to functions and i come across a limit which i cannot understand.The question is If $a$ and $b$ are positive real numbers such that $a-b=2,$ then find the smallest value of the constant $L$ for which $\sqrt{x^2+ax}-\sqrt{x^2+bx}<L$ for all $x>0$ First i found ...

0
Q: Continuum hypothesis outside of ZFC

mickMy understanding is that it is impossible to prove and disprove the Continuum hypothesis in ZFC. Would it be possible to prove or disprove it in some other axiomatic set theory?

0
Q: Prove [(q ˄ (p ↔ ¬ q) ) → q] is a tautology using logic laws

J.doe123How to prove that this statement is tautology using logic laws (q ˄ (p ↔ ¬ q) ) → q I got stuck here: (q ˄ (p ↔ ¬ q) ) → q -(q V (p ↔ ¬ q) ) → -q -q V -(p <-> -q) -> -q

Short question. [Prove [(q ˄ (p ↔ ¬ q) ) → q] is a tautology using logic laws](math.stackexchange.com/q/1576848)
 
2:55 PM
0
Q: Some of the most useful questions/answers on stack overflow are for disallowed questions

xaxxonWhy is "using namespace std;" considered bad practice? For just one example, should probably be closed as "primarily opinion based", but there are good subtle intricacies to the answers here that many people might not know about like in "sbi"'s answer. There may be a more appropriate place for ...

0
Q: Merge [vis] and [vis.js]

AlmisBoth tags referring to the same javascript library which can be found here.

 
0
Q: Did the OEIS skip some $n$ re the "congruent number problem"?

Tito Piezas III(This post made me re-visit congruent numbers.) I. Wikipedia: An integer $n$ is a congruent number if there are rationals $a,b,c$ such that, $$a^2+b^2 = c^2\\ \tfrac{1}{2}ab = n$$ or, alternatively, the elliptic curve, $$x^3-n^2x = y^2\tag1$$ is solvable in the rationals. II. Mathworld and ...

0
Q: How to solve equation with regular singularity problem

RajabGood day, Here is my equation $$ (x-1)^2 (x-h)^2q_1(x,\Omega)y''(x)+(x-1)(x-h)q_2(x,\Omega)y'(x)+q_3(x,\Omega)y(x)=0 $$ ( with $q_1(x,\Omega)$[polynomial function 38 degree], $q_2(x,\Omega)$ [polynomial function 39 degree], $q_3(x,\Omega)$ [polynomial function 40 degree] regular fu...

0
Q: How do you select base-cases for this proof?

lucidgold Let P(n) be the statement that a postage of n cents can be formed using just 4-cent and 7-cent stamps. Show by mathematical induction that P(n) is true for n ≥ 18. Hint: carefully determine what the base cases are. Solution: Base cases: P(18) is true as 18 = 4 + 2 ∗ 7. P(19) is tru...

0
Q: Lie group question: If $\gamma^{-1}\dot{\gamma}\in\mathfrak{g}$ everywhere, does $\gamma(t)\in G$?

user298909Let $G$ be a Lie subgroup of $GL(n,\Bbb R)$ and $\mathfrak{g}\subseteq M(n,\Bbb R)$ its Lie algebra. Suppose that we have a smooth curve $$\gamma:\Bbb R\to G$$ with $\gamma(0)=I$. Then, it induces a curve $$\alpha:\Bbb R\to \mathfrak{g},\quad \alpha(t)=\gamma(t)^{-1}\frac{d\gamma}{dt}(t).$$ I am ...

This site uses MathJax formatting of formulas. More tips here. (autocomment)Normal Human 21 secs ago
0
Q: $f:\mathbb R \to \mathbb R$ be differentiable such that $f(0)=0$ and $f'(x)>f(x),\forall x \in \mathbb R$ ; then is $f(x)>0,\forall x>0$?

Saun DevLet $f:\mathbb R \to \mathbb R$ be a differentiable function such that $f(0)=0$ and $f'(x)>f(x),\forall x \in \mathbb R$ ; then is it true that $f(x)>0,\forall x>0$ ?

0
Q: A question about boundedness of operators on function spaces

bjk1806Let $\Omega\subset\mathbb{R}^n$ and $T$ is an operator (For example H-L maximal operator $M$, Calderon-Zygmund operator $K$ etc.) $$ (1)... \|Tf\|_{L_p(\mathbb{R}^n)}\leq C \|f\|_{L_p(\mathbb{R}^n)} $$ and $$ (2)... \|Tf\|_{L_p(\Omega)}\leq C \|f\|_{L_p(\Omega)}. $$ Can we say anything about $(1)...

0
Q: limit of a sequence zero?

user159452Let two points $a,b\in\mathbb{S}^{2}$, where $\mathbb{S}^{2}$ is the two-dimensional simplex in $R^{3}$ with $\sum{}x_{i}=1$ for all $x\in\mathbb{S}^{2}$. $x_{1},x_{2},x_{3}$ are the coordinates of $x$. The Kullback-Leibler divergence is defined as follows: $$ D_{\mbox{KL}}(b,a)=\sum_{i=1}^{3}b...

0
Q: $\mathrm{Tr}_{L/K}(x^i/g'(x))\in R$?

mqxLet $R$ be a Dedekind domain, with fraction field $K$, let $L/K$ be a separable extension generated by one element $x$, let $g$ be the minimal polynomial of $x$ over $K$. Do we have $\mathrm{Tr}_{L/K}(x^i/g'(x))\in R$?

0
Q: Combinatorial expression for all ternary strings that don't have consecutive 1's and 2's

I. VajsThe problem is: find $C_n$ which is the number of all ternary strings (length $n≥0$) that don't have consecutive 1's and 2's using combinatorics. Any tips or advices?

 
3:14 PM
-2
Q: Reopen my question

VaindilI'm sorry if this shouldn't be asked here, I'm not familiar with specifically what type of questions can be asked on meta.SO. Yes, I read the meta info page. Can I get this question reopened? I didn't initially clarify like I should have, but the problem is that those steps did not fix my problem.

 
0
Q: >Integrate $\displaystyle \int{\dfrac{x}{1+x^4}}dx$.

Puzzled417 Integrate $\displaystyle \int{\dfrac{x}{1+x^4}}dx$. The best way I can think of doing this is by breaking $\dfrac{x}{1+x^4}$ into partial fractions but that would be messy.

Tall formulas in titles break the layout of question lists. Please replace \dfrac with \frac, remove \displaystyle in the title. (from a bot)Normal Human 21 secs ago
0
Q: Sigma-finiteness and absolutely continuous measures

TheOscillatorSuppose we have an abstract $\sigma$-finite measure space $(X,\mathscr{A},\mu)$ and let $\nu$ be another measure on $(X,\mathscr{A})$ aswell, with the property that $\nu <<\mu$, i.e $\nu$ is absolutely continuous w.r.t $\mu$ . Does it necessarily imply that $\nu$ is $\sigma$-finite on $(X,\math...

0
Q: Derivative of inverse matrix

Charles ChouSuppose $\Omega \left( \mathbf{\alpha }\right) $ is a $T\times T$ full rank matrix where $\mathbf{\alpha }$ is a $p\times 1$ vector, then what's the exact expression for $\frac{\partial \Omega ^{-1}\left( \mathbf{\alpha }% \right) }{\partial \mathbf{\alpha }^{\prime }}?$ If $\mathbf{\alpha }$ is ...

0
Q: Trigonometric function on modulus function

AyushakjIf $sin x +cos x = a$ Then how do I get value of modulus of $sin x - cos x$, value in terms of $a$

0
Q: Numerical method for SDEs

gerdI'm using a 4th order Adams predictor-corrector method to numerically solve a regular differential equation. Now I would be interested to be able to include a noisy term to the equation -as in the Euler-Maruyama method, the classical and easy way to simulate a Brownian motion via a Wiener process...

0
Q: exponential equation... I do not know how to this HW

RA3SKtoday I have big problem. Our teacher gave us this HW with exponential equation for marks. I do not want to get bad mark so I am here. Please help me. I did a lot. And I also know how to solve basic exponential equation problems, but not these. Please help me. enter image description here

 
3:33 PM
0
Q: How to better organize your Stackoverflow questions?

smwikipediaThrough the years, I have been asking many questions on StackOverflow. I kind of consider these questions as my personal knowledge base. But I found it's kind of inconvenient to do the following things with my threads (both my own posts and my favorites): Archive them Search them Classify them...

 
0
Q: How can I EXPLAIN this intuitively?

YiFeiSay I have a ODE: $f'(x)=-kf(x)$ $f(0)=a$, how can I explain that if $a \not= 0$ then never can f(x)=0 for all x. (Please do not solve the equation explicitly)

Short question. Question contains please. How can I EXPLAIN this intuitively?
0
Q: $R$ be a commutative ring with unity having at most $5$ distinct ideals (including $\{0\}$ and $R$ itself) ; then is it true that $R$ is PIR ??

Saun DevLet $R$ be a commutative ring with unity having at most $5$ distinct ideals (including $\{0\}$ and $R$ itself) ; then is it true that $R$ is a principal ideal ring i.e. is every ideal of $R$ principal ?

0
Q: Path-connectedness of singletons (question about Munkres)

AvatrinIn corollary 59.2, Munkres states: Suppose $X = U \cup V$, where U and V are open sets of X; suppose $U \cap V$ is nonempty and path connected. If U and V are simply connected, then X is simply connected. Then comes exercise one. As a note, Munkres states that the union of two simply connected ...

Words such as question are uninformative in titles. Please edit the title so that it better describes the specifics of your question. Do not hesitate to make it longer or include a formula if needed. More tips here. (autocomment)Normal Human 21 secs ago
0
Q: Equation $(x+y\sqrt{2})^6+(u+v\sqrt{2})^6=5+7\sqrt{2}$ in $\mathbb{Q}$

Raheem NajibSolve equation $$(x+y\sqrt{2})^6+(u+v\sqrt{2})^6=5+7\sqrt{2}$$ for $x,y,u,v$ rationals. Sorry but I have not any idea.

0
Q: Can't Simplify this equation for a Ellipse(Complex Numbers)

Teyash ArjunI'm asked to sketch the subset for: {z ∈ C : |z + i| + |z + 1| = 2} I've gotten to the point where I've got the modulus form of |z + i| + |z + 1|: sqrt(x^2+(y+1)^2) + sqrt((x+1)^2+y^2) = 2 How do I change this to the equation for an ellipse ?

This site uses MathJax formatting of formulas. More tips here. (from a bot)Normal Human 21 secs ago
0
Q: Id-T is a surjective operator

AlbertI m sorry . This question is so primary. T is a bounded linear operator on U and U is a Banach space. (||T||<1) then id-T is a surjective operator.

Short title. Short question. Id-T is a surjective operator
0
Q: List an utility function's elements

CrushedPixelS is a finite set, and v: S → ℕ is a valuation function. We have S := {0,1,2} and the valuation function v v: S → ℕ, k → 2k+1 The task is to list all values for v's utility function u: utility function u How would I go on about doing so? We weren't introduced to the concept of valuation ...

Questions tend to get more attention when they have a tag for a broad area of mathematics relevant to the question. Some of these tags might fit. (from a bot)Normal Human 21 secs ago
 
3:55 PM
0
Q: Example of integrally closed

SelvaLet $g,h \in k[x,y]$ such that $\sqrt(h,g)=(x,y)$. Is $k[x,y] \backslash (f^2+h^2)$ integrally closed?

Short title. Short question. Example of integrally closed
0
Q: Derivative of norm 1

tophitLet $A\in \mathcal{R}^{nxn} $ be a real matrix and $x_0,x\in \mathcal{R}^{nx1} $. What is the gradient of the following function at $x$ assuming $Ax \neq x_0$: $||Ax-x_0||_1$ thanks

Short title. Title ends with a digit. Derivative of norm 1
 
00:00 - 16:0016:00 - 00:00

« first day (43 days earlier)      last day (531 days later) »